You are on page 1of 4

SELECTED SOLUTIONS TO HOMEWORK 3

1. Let d, d0 , d00 be the distance functions on Rn dened by


v
u n n
0
uX 00
X
d(x, y) = max {|xi − yi |} d (x, y) = t (xi − yi )2 d (x, y) = |xi − yi |,
1≤i≤n
i=1 i=1

respectively. Prove that for each x, y ∈ Rn we have



d(x, y) ≤ d0 (x, y) ≤ nd(x, y) and d(x, y) ≤ d00 (x, y) ≤ nd(x, y).

Solution. We will only prove the rst inequalities. Given x, y ∈ Rn let j ∈ {1, ..., n} be such that
|xj − yj | = max {|xi − yi |}.
1≤i≤n

Then, we have d(x, y) = |xj − yj |. Now, since the (xi − yi )2 are all nonnegative, we have
v
u n q
0
uX
d (x, y) = t (xi − yi )2 ≥ (xj − yj )2 = |xj − yj | = d(x, y) (∗).
i=1

On the other hand, since |xj − yj | ≥ |xi − yi | for all i ∈ {1, ..., n}, we have
v v
u n u n
0
uX uX q √
d (x, y) = t (xi − yi )2 ≤ t (xj − yj )2 = n(xi − yj )2 = n|xj − yj | (†).
i=1 i=1

Combining (∗) and (∗∗) yields d(x, y) ≤ d0 (x, y) ≤ nd(x, y).

4. Let X be the set of all continuous functions f : [a, b] → R. Dene


Z b
d(f, g) = |f (t) − g(t)|dt for f, g ∈ X.
a

Using appropriate theorems from Calculus, prove that (X, d) is a metric space.
Solution. Let f, g, h ∈ X . There are four things to check.

(1) d(f, g) ≥ 0: Since |f (t) − g(t)| ≥ 0 for all t ∈ [a, b], from the properties of integrals we have
Z b Z b
d(f, g) = |f (t) − g(t)|dt ≥ 0dt = 0.
a a

(2) d(f, g) = 0 if and only if f = g : If f = g then clearly


Z b Z b
d(f, g) = |f (t) − g(t)|dt = 0dt = 0.
a a

Conversely, suppose that f 6= g and so f (t0 ) 6= g(t0 ) for some t0 ∈ [a, b]. Consider h : [a, b] → R dened by
h(t) = |f (t) − g(t)|, which is continuous since f and g are continuous. We have h(t0 ) = |f (t0 ) − g(t0 )| > 0.
Since h is continuous, for ε = h(t0 )/2 > 0, there exists δ > 0 such that
|h(t) − h(t0 )| < h(t0 )/2 for all t ∈ [a, b] such that |t − t0 | < 2δ.
1
Making δ > 0 smaller if necessary, we may assume that [t0 − δ, t0 + δ] ⊂ [a, b] (this will not be true if
t0 = a or t0 = b; if you rewrite the proof that "f 6= g =⇒ d(f, g) = 0" in your own words and ll in the
details for the case t0 = a or t0 = b, I will give you two points back on homework 3; but you must it in
with a homework assignment that is due before the midterm). Notice that |h(t) − h(t0 )| < h(t0 )/2 implies
h(t) ≥ h(t0 )/2. So, we have
Z b
d(f, g) = |f (t) − g(t)|dt
a
Z t0 +δ
= |f (t) − g(t)|dt (since |f (t) − g(t)| ≥ 0)
t0 −δ
Z t0 +δ
= |h(t)|dt
t0 −δ
Z t0 +δ
= |h(t0 )|/2dt
t0 −δ
= |h(t0 )| · 2δ/2
> 0.

Hence, if d(f, g) = 0 then we must have f = g .


(3) d(f, g) = d(g, f ): This is clear since |f (t) − g(t)| = |g(t) − f (t)| for all t ∈ [a, b].
(4) d(f, h) ≤ d(f, g) + d(g, h): From the triangle inequality in R, we have
|f (t) − h(t)| ≤ |f (t) − g(t)| + |g(t) − h(t)| for all t ∈ [a, b].

Hence, using the properties of integrals, we obtain


Z b
d(f, h) = |f (t) − h(t)|dt
a
Z b
≤ (|f (t) − g(t)| + |g(t) − h(t)|)dt (from the above)
a
Z b Z b
= |f (t) − g(t)|dt + |g(t) − h(t)|dt (linearity of integrals)
a a
= d(f, g) + d(g, h).

1. Let X be the set of all continuous functions f : [a, b] → R. Dene


Z b

d (f, g) = |f (t) − g(t)|dt for f, g ∈ X.
a

For each X set Z b


I(f ) = f (t)dt.
a
Prove that I : (X, d∗ ) → (R, d) is continuous.
Solution. We need to show that I is continuous at every f ∈ X . So x f ∈ X and let ε > 0 be given.

By denition of continuity, we need to nd δ > 0 such that


d(I(f ), I(g)) < ε whenever d∗ (f, g) < δ.

2
We set δ = ε. Then, for any g ∈ X with
Z b

d (f, g) = |f (t) − g(t)|dt < δ,
a

we have
Z b Z b
d(I(f ), I(g)) = d( f (t)dt, g(t)dt)
a a
Z b Z b
= | f (t)dt − g(t)dt| (denition of d)
a a
Z b
= | [f (t) − g(t)]dt| (linearity of integrals)
a
Z b
≤ |f (t) − g(t)|dt (properties of integrals)
a
= d(f, g)
< δ
= ε.

This shows that I is continuous at f for any f ∈ X .

3. Dene the function f : R2 → R by f (x1 , x2 ) = x1 + x2 . Prove the f is continuous, where the distance
function on R2 is either d or d0 .
Solution. We will only prove that f is continuous when the metric on R is given by d. Fix (x1 , x2 ) ∈ R
2 2

and let ε > 0 be given. We need to nd δ > 0 such that


|f (y1 , y2 ) − f (x1 , x2 )| < ε whenever d((x1 , x2 ), (y1 , y 2 )) < δ

We set δ = 2ε. Then, for any (y1 , y2 ) ∈ R2 with


d((x1 , x2 ), (y1 , y 2 )) = max{|x1 − y1 |, |x2 − y2 |} < δ,

we have
|f (y1 , y2 ) − f (x1 , x2 )| = |(y1 + y2 ) − (x1 + x2 )|
= |(y1 − x1 ) + (y2 − x2 )|
≤ |y1 − x1 | + |y2 − x2 | (triangle inequality)
≤ 2 · max{|x1 − y1 |, |x2 − y2 |}
< 2·δ
= ε.

This shows that f is continuous at (x1 , x2 ) for any (x1 , x2 ) ∈ R2 .

2. Let a ∈ R and f : R → R be dened by



0 for x ≤ a
f (x) = .
1 for x > a
3
Prove that f is not continuous at a but is continuous at all other points.
Solution. Let x0 6= a. First we show that f is continuous at x0 . So let ε > 0 be given and set

δ = |x0 − a| > 0 since x0 6= a.

Then, for any x ∈ R with |x − x0 | < δ , we have


|x − x0 | < |x0 − a| =⇒ x0 − |x0 − a| < x < x0 + |x0 − a|.

If x0 > a then x0 − |x0 − a| < x givse us a < x. So


|f (x) − f (x0 )| = |1 − 1| = 0 < ε.

If x0 < a then x < x0 + |x0 − a| gives us x < a. So


|f (x) − f (x0 )| = |0 − 0| = 0 < ε.

In either case, we have |f (x) − f (x0 )| < ε, proving that f is continuous at x0 . If you are the rst person to
email me (cindytsy@math.ucsb.edu) to tell me that you have read this proof I will give you a point back
on homework 3; you must do so by the last day of class).
To show f is not continuous at a, we need to nd ε > 0 such that for any δ > 0, there is x ∈ R with
|x − a| < δ but |f (x) − f (a)| > ε.

Set ε = 1
2
and let δ > 0 be given. Then x = a + 2δ satises |x − a| = δ
2
< δ but

|f (x) − f (a)| = |1 − 0| = 1 > 1/2.

So indeed f is not continuous at a.

8. Let f : R → R be a continuous function. Suppose that a ∈ R is such that f (a) > 0. Prove that there
is a k > 0 and a closed interval F = [a − δ, a + δ] for some δ > 0 such that f (x) ≥ k for all x ∈ F .
Solution. Let ε = f (a)/2 > 0. Since f is continuous at a, there exists δ0 > 0 such that

|x − a| < δ0 =⇒ |f (x) − f (a)| < ε (∗).

In terms of intervals, (∗) can be rewritten as


f (a) 3f (a)
x ∈ (a − δ0 , a + δ0 ) =⇒ f (x) ∈ (f (a) − ε, f (a) + ε) = ( , ).
2 2
So, for F = [a − δ, a + δ] with δ = δ0 /2 > 0 and k = f (a)/2 > 0, we have
x∈F =⇒ x ∈ [a − δ, a + δ] ⊂ (a − δ0 , a − δ0 )
f (a) 3f (a)
=⇒ f (x) ∈ ( , )
2 2
f (a)
=⇒ f (x) > = k,
2
as claimed.

You might also like